10
$\begingroup$

I was reading Auslander's talk at the 1962 ICM (beginning of Section 2 on this page). At the end, the reference began:

[1] M. Auslander, Modules over unramified regular local rings, Illinois. J. Math. 5 (1961), pp. 631–647.

[2] M. Auslander, Modules over unramified regular local rings II, Illinois. J. Math. (To appear).

The trouble is, I can not find [2] on MathSciNet or Google. I also looked at his Selected works without luck. Of course, there are many papers that are forthcoming but never materialized, but this one even has the journal name attached to it! So

What happened to Auslander paper [2] above?

Note that [1] is an influential paper (still being quoted very recently). I enjoyed it a lot, and would really like to read [2].

UPDATE: both Professor Buchsbaum and Professor Reiten have graciously replied to my query about this missing paper. Unfortunately, neither of them know what happened.

$\endgroup$
4
  • $\begingroup$ The only person I can think of who might have direct knowledge is David Buchsbaum. $\endgroup$ Aug 6, 2011 at 13:47
  • $\begingroup$ @Graham: good point. May be Idun Reiten as well? Perhaps I should try emailing them. $\endgroup$ Aug 6, 2011 at 14:30
  • $\begingroup$ Idun was a child back then -- she got her PhD in 1971. I think emailing David is probably the best bet. $\endgroup$ Aug 6, 2011 at 15:17
  • $\begingroup$ @Graham: Yes, but their collaboration is longer. Also, she edited his Selected Works. I just emailed both, let's see. $\endgroup$ Aug 6, 2011 at 15:29

1 Answer 1

2
$\begingroup$

His "Selected Works" (AMS: http://www.ams.org/bookstore-getitem/item=CWORKS-10) lists on Chapter II, two articles with the same title "Modules over unramified regular local rings"

$\endgroup$
4
  • 1
    $\begingroup$ Yes, the second one is a reprint of his ICM talk. $\endgroup$ Aug 6, 2011 at 2:00
  • $\begingroup$ @Zaldiva: compare that with Auslander's ICM talk linked above: books.google.com/… $\endgroup$ Aug 6, 2011 at 2:44
  • 3
    $\begingroup$ @Zaldiva: but thank you for trying. $\endgroup$ Aug 6, 2011 at 3:25
  • $\begingroup$ I will accept this answer for now to prevent the question from being bumped to the top frequently. If anyone knows what happened, please add your answers. $\endgroup$ Aug 27, 2011 at 0:57

Your Answer

By clicking “Post Your Answer”, you agree to our terms of service and acknowledge you have read our privacy policy.

Not the answer you're looking for? Browse other questions tagged or ask your own question.